Clinical scenario and Handover 1 Flashcards

1
Q

You are a medical SHO on the acute take overnight. You have been referred a 72-year-old woman from A+E with persistent dizziness. You can see from her record that she has been seen in A+E three times already this week with the same issue. She has a past medical history of coronary vascular disease, type 2 diabetes and hypertension.

Her observations in A+E are as follows:

BP: 172/95
HR: 105
RR: 19
Sats: 97% RA
T: 37.1

What is most important to cover in hx and ex?

A

Start with A-E
Then once stable and happy.

Ask more to pinpoint dizziness - vertigo or light headedness, when it comes on, any associated chest pain, SOB, palpitations, N+V.

Ask more about timeline of symptoms and elevating or exacerbating factors.

On examination - listen to chest, assess hydration status, L/S BP, neuro exam - cranial nerves and cerebellar function.

How well did you know this?
1
Not at all
2
3
4
5
Perfectly
2
Q

You are a medical SHO on the acute take overnight. You have been referred a 72-year-old woman from A+E with persistent dizziness. You can see from her record that she has been seen in A+E three times already this week with the same issue. She has a past medical history of coronary vascular disease, type 2 diabetes and hypertension.

Her observations in A+E are as follows:

BP: 172/95
HR: 105
RR: 19
Sats: 97% RA
T: 37.1

When taking a history from someone with dizziness are there any features that would concern you that may be indicative of something more serious happening and is there anything in this case that would make you want to investigate further?

A

Worsening dizziness - atypical for vertigo

Associated chest pain or palpitations with the dizziness

Any neuro signs such as diplopia or nystagmus suggestive of cranial cause

How well did you know this?
1
Not at all
2
3
4
5
Perfectly
3
Q

You are a medical SHO on the acute take overnight. You have been referred a 72-year-old woman from A+E with persistent dizziness. You can see from her record that she has been seen in A+E three times already this week with the same issue. She has a past medical history of coronary vascular disease, type 2 diabetes and hypertension.

Her observations in A+E are as follows:

BP: 172/95
HR: 105
RR: 19
Sats: 97% RA
T: 37.1

What signs/symptoms would be concerning for a cerebellar cause?

A

DANISH-

Dysdiadokinesia
Ataxia
Nystagmus
Intention tremor
Speech disturbance
Hypotonia

How well did you know this?
1
Not at all
2
3
4
5
Perfectly
4
Q

You are a medical SHO on the acute take overnight. You have been referred a 72-year-old woman from A+E with persistent dizziness. You can see from her record that she has been seen in A+E three times already this week with the same issue. She has a past medical history of coronary vascular disease, type 2 diabetes and hypertension.

Her observations in A+E are as follows:

BP: 172/95
HR: 105
RR: 19
Sats: 97% RA
T: 37.1

From your history and examination, you find out that the dizziness has been persistent and present for four days. The patient feels it has been getting worse, and she has had one fall at home when she fell to the left side. Additionally, she has a headache, which is not responding to medication. She has a history of TIA in the past in addition to her other cardiovascular history. What is the most likely diagnosis, and why do you think this and what other differentials would you include?

A

Need to R/O posterior circulatory stroke especially with CV and TIA Hx.

Need to try and think of cause especially due to 3rd presentation

Other differentials - migraine or intrcranial lesion

How well did you know this?
1
Not at all
2
3
4
5
Perfectly
5
Q

You are a medical SHO on the acute take overnight. You have been referred a 72-year-old woman from A+E with persistent dizziness. You can see from her record that she has been seen in A+E three times already this week with the same issue. She has a past medical history of coronary vascular disease, type 2 diabetes and hypertension.

Her observations in A+E are as follows:

BP: 172/95
HR: 105
RR: 19
Sats: 97% RA
T: 37.1

What investigations would you want to preform?

A

Always give a reason for investigations -

Routine bloods - FBC, CRP, LFTs U+Es (look for electrolyte abnormalities, FBC to look at anaemia/infection as with CRP, LFTs to R/O alcoholism)

Coag and INR - to reverse any coagulopathy if needs intervention for intracranial haemorrhage.

CT head - useful in ruling out an intracranial haemorrhage. It may also pick up an infarct if it is established

MRI head - once stable. To look for area of ischaemia, especially if in cerebellum.

L/S BP - To rule out postural hypotension

How well did you know this?
1
Not at all
2
3
4
5
Perfectly
6
Q

You are a medical SHO on the acute take overnight. You have been referred a 72-year-old woman from A+E with persistent dizziness. You can see from her record that she has been seen in A+E three times already this week with the same issue. She has a past medical history of coronary vascular disease, type 2 diabetes and hypertension.

Her observations in A+E are as follows:

BP: 172/95
HR: 105
RR: 19
Sats: 97% RA
T: 37.1

What is the treatment for acute stroke?

A

Mostly dependent on timing of presentation since onset of symptoms and the type of stroke.

Need urgent CT head first to determine if ischaemic or haemorrhagic.

If haemorrhagic - reverse anticoagulation and refer urgently to neurosurgery.

If ischaemic - 300mg of aspirin (2 weeks) and thrombolysis if <4.5hrs and thrombectomy if <24hrs

Get aspirin and clopidogrel for 2 weeks then clopidogrel life long afterwards.

Discuss cases with stroke team BEFORE initiating treatment

How well did you know this?
1
Not at all
2
3
4
5
Perfectly
7
Q

You are a medical SHO on the acute take overnight. You have been referred a 72-year-old woman from A+E with persistent dizziness. You can see from her record that she has been seen in A+E three times already this week with the same issue. She has a past medical history of coronary vascular disease, type 2 diabetes and hypertension.

Her observations in A+E are as follows:

BP: 172/95
HR: 105
RR: 19
Sats: 97% RA
T: 37.1

How would you consider anticoagulation in a patient with NEW AF that presents with stroke

A

Treat as normal stroke then switch to DOAC after 10-14 days. This is due to increase risk of intracranial haemorrhage.

How well did you know this?
1
Not at all
2
3
4
5
Perfectly
8
Q

You are a medical SHO on the acute take overnight. You have been referred a 72-year-old woman from A+E with persistent dizziness. You can see from her record that she has been seen in A+E three times already this week with the same issue. She has a past medical history of coronary vascular disease, type 2 diabetes and hypertension.

Her observations in A+E are as follows:

BP: 172/95
HR: 105
RR: 19
Sats: 97% RA
T: 37.1

How are acute stroke services run in the UK?

A

Usually via HASU (Hyperacute stroke units) with teams of consultants, registrars and nurses.

Not all hospitals have a HASU so may need transferred.

How well did you know this?
1
Not at all
2
3
4
5
Perfectly
9
Q

You are called to see a patient on your acute medical ward who has suddenly developed left-sided hemiparesis and facial droop. What should you do?

A

Urgent assessment - A-E and immediately call stroke services and order CT head

Transfer then get imaging over there if no HASU at your hospital as time is brain.

How well did you know this?
1
Not at all
2
3
4
5
Perfectly
10
Q

You are a medical SHO on the acute take overnight. You have been referred a 72-year-old woman from A+E with persistent dizziness. You can see from her record that she has been seen in A+E three times already this week with the same issue. She has a past medical history of coronary vascular disease, type 2 diabetes and hypertension.

Her observations in A+E are as follows:

BP: 172/95
HR: 105
RR: 19
Sats: 97% RA
T: 37.1

The relatives of the patient admitted with the posterior circulation stroke have arrived. When you see them, they become extremely upset when you tell them their mother has had a stroke; they want to make a complaint because this wasn’t picked up on her initial visit to A+E earlier in the week. How would you manage this?

A

Important to empathise with patient and family and explain steps taken so far for management and review of stroke.

Aknowledge their right to complain and offer PALS.

Relfect and document on this.

How well did you know this?
1
Not at all
2
3
4
5
Perfectly
11
Q

The patient is on her way to the scanner, and it is the end of your shift. You now have one minute to handover the patient in this scenario to your registrar/consultant as if you were at the Acute Medical Handover.

A

Situation: This is a 72 year old woman who has presented with a four day history of dizziness and unilateral weakness, suspicious for a posterior circulation stroke.

Background: She has risk factors for stroke including diabetes, hypertension and previous TIAs.

Assessment: She was hypertensive on admission and symptoms have persisted for several days in which she has had several admissions. I have arranged an urgent CT head, which she is currently on her way to.

Recommendations: Please chase the results of the CT head and then discuss with the local stroke team. She is outside of the thrombolysis window but following the scan, if there is no evidence of acute intracranial haemorrhage, the patient could be commenced on high dose aspirin. She will need investigations to try and establish the cause of any stroke and then an MRI if appropriate to confirm the diagnosis.

How well did you know this?
1
Not at all
2
3
4
5
Perfectly
12
Q

You are the medical SHO on-call overnight. You have been asked to see a 31-year-old man on the gastroenterology ward. He had been admitted with abdominal pain and temperature 4 days ago. You have been asked to see him urgently by the senior nurse because he is coughing, his saturations are low, he is having difficulty breathing and he has a high temperature. The nurse tells you that the patient is in a side room. His observations are as follows:

BP 97/67
HR 135
Sats 92% on 4L of oxygen
RR 26
T 38.7

What is the most likely diagnosis?

A

Sepsis - chest source due to low BP and tachycardia. Likely compensating

Could be HAP due to length of time in hospital.

Could be COVID/Flu - particularly bad this year.

How well did you know this?
1
Not at all
2
3
4
5
Perfectly
13
Q

You are the medical SHO on-call overnight. You have been asked to see a 31-year-old man on the gastroenterology ward. He had been admitted with abdominal pain and temperature 4 days ago. You have been asked to see him urgently by the senior nurse because he is coughing, his saturations are low, he is having difficulty breathing and he has a high temperature. The nurse tells you that the patient is in a side room. His observations are as follows:

BP 97/67
HR 135
Sats 92% on 4L of oxygen
RR 26
T 38.7

How would you assess the patient?

A

First - Get appropriate PPI on to protect myself at to stop the spread of a potential communitable diease.

A-E assessment, go through fully to get the marks. Thinking about other investigations also.

A - make sure pt can talk to me. If not check for signs of life and shout for someone to call 2222 if airway concerns/cardiac arrest.

B - Patient has low sats and raised RR. Need to listen to chest and check chest expansion and percussion. Look for poor resp effort to R/O resp distress. Do ABG to check for resp failure. Order CXR.

C - can see some degree of compensation to septic shock. Will check CRT, pulse to see if regular and volume, listen to heart and look at urine output. I would then get ECG and insert large bore cannula and take off bloods - FBC, CRP, LFTs, U+Es, Lactate, blood cultures. Then give IVT bolus to see if fluid responsive.

D - Check glucose and GCS. Paying particular attention to see if pt confused.

E - full head to toe examination to look for skin rashes, focal neurology, leg swellings suggestive of DVT/PE. Recheck temperature. May give paracetamol to help with this. Take viral swabs also.

How well did you know this?
1
Not at all
2
3
4
5
Perfectly
14
Q

You are the medical SHO on-call overnight. You have been asked to see a 31-year-old man on the gastroenterology ward. He had been admitted with abdominal pain and temperature 4 days ago. You have been asked to see him urgently by the senior nurse because he is coughing, his saturations are low, he is having difficulty breathing and he has a high temperature. The nurse tells you that the patient is in a side room. His observations are as follows:

BP 97/67
HR 135
Sats 92% on 4L of oxygen
RR 26
T 38.7

Should you wear PPE to see this patient?

A

Would depend on trust specific guidelines. However, due to high clinical suspscion of resp infection - would consider this to limit risk to myself and others then would be guided by nursing team once swab results etc back.

How well did you know this?
1
Not at all
2
3
4
5
Perfectly
15
Q

You are the medical SHO on-call overnight. You have been asked to see a 31-year-old man on the gastroenterology ward. He had been admitted with abdominal pain and temperature 4 days ago. You have been asked to see him urgently by the senior nurse because he is coughing, his saturations are low, he is having difficulty breathing and he has a high temperature. The nurse tells you that the patient is in a side room. His observations are as follows:

BP 97/67
HR 135
Sats 92% on 4L of oxygen
RR 26
T 38.7

Would this warrant escalaltion

A

Although I feel confident in managing acutely unwell patients. I feel that once I have done an initial A-E and done intial management - this would warrant escalation as he is young and this is septic shock.

To escalate to reg and possibly crit care.

May need ITU/HDU for non-invasive ventilation so important to be aware of this.

How well did you know this?
1
Not at all
2
3
4
5
Perfectly
16
Q

You are the medical SHO on-call overnight. You have been asked to see a 31-year-old man on the gastroenterology ward. He had been admitted with abdominal pain and temperature 4 days ago. You have been asked to see him urgently by the senior nurse because he is coughing, his saturations are low, he is having difficulty breathing and he has a high temperature. The nurse tells you that the patient is in a side room. His observations are as follows:

BP 97/67
HR 135
Sats 92% on 4L of oxygen
RR 26
T 38.7

This patient has been admitted under the gastroenterology team for management of his Crohn’s disease and is on long term immunosuppression. Would you add anything to your suspected diagnosis or your management?

A

Important to know that immunosuppression can increase risk of

  • opportunistic infections like TB and PJP.
  • severe resp complications like empyema

I would discuss with on-call micro to determine if they want me to put the patient on different broad-spectrum abxs.

How well did you know this?
1
Not at all
2
3
4
5
Perfectly
17
Q

You are the medical SHO on-call overnight. You have been asked to see a 31-year-old man on the gastroenterology ward. He had been admitted with abdominal pain and temperature 4 days ago. You have been asked to see him urgently by the senior nurse because he is coughing, his saturations are low, he is having difficulty breathing and he has a high temperature. The nurse tells you that the patient is in a side room. His observations are as follows:

BP 97/67
HR 135
Sats 92% on 4L of oxygen
RR 26
T 38.7

Give an SBAR

A

Situation: This 31 year old man with a past medical history of Crohn’s disease is presenting with signs and symptoms consistent with sepsis, likely respiratory in origin.

Background: He is on long term immunosuppression which makes him more at risk from hospital acquired infections, which would also include influenza and COVID-19 pneumonitis.

Assessment: I have started fluid resuscitation and treatment with broad-spectrum antibiotics. He is on high flow oxygen.

Recommendations: This man needs an urgent senior clinical review and may need escalation of treatment to HDU or ITU for respiratory support. I will ask for the ITU team to review him. His background of immunosuppression means I think we also need to consider atypical sources of infection, including intra-abdominal collections and empyema, and would like the advice of the microbiology team on appropriate antimicrobials.

How well did you know this?
1
Not at all
2
3
4
5
Perfectly
17
Q

What treatment options are available for COVID-19?

A

Depends on severity.

Supportive management - fluids, analgesia, observations, maintaining sats

If on o2 - dexamethasone, paxlovid if immune suppressed.

Resp support and other organ support - NIV etc.

How well did you know this?
1
Not at all
2
3
4
5
Perfectly
18
Q

You are the SHO in Ambulatory Care. You are asked to see Mrs Jacobs, a 56-year-old lady who has been referred in by her GP with a painful left calf. You are asked to review her and organise the appropriate investigations. She has noticed her left leg has become painful and swollen over the last few days. She has no past medical history that the GP is aware of.

What would you be thinking about in your differential and what would you like to ask in the patient’s history to help with this?

A

Top differential - DVT

Others - cellulitis, limb ischaemia, trauma, referred joint pain.

Will ask about timeline of pain, its character, elevating or worsening factors.

Recent DVT risk factors - long travel, cancer, COCP/HRT, recent surgery, coagulopathy disorders.

Ensure no PE signs

How well did you know this?
1
Not at all
2
3
4
5
Perfectly
19
Q

You are the SHO in Ambulatory Care. You are asked to see Mrs Jacobs, a 56-year-old lady who has been referred in by her GP with a painful left calf. You are asked to review her and organise the appropriate investigations. She has noticed her left leg has become painful and swollen over the last few days. She has no past medical history that the GP is aware of.

What would you be looking for on exam?

A

A-E

Ensure no signs of resp distress or CP.

Assess for erythema, temp, site of pain, palpation of leg and examination of ankle and knee joints. Looking and palpated for peripheral oedema. Check peripheral pulses. Always comparing both legs.

Compare this to Wells score.

How well did you know this?
1
Not at all
2
3
4
5
Perfectly
20
Q

You are the SHO in Ambulatory Care. You are asked to see Mrs Jacobs, a 56-year-old lady who has been referred in by her GP with a painful left calf. You are asked to review her and organise the appropriate investigations. She has noticed her left leg has become painful and swollen over the last few days. She has no past medical history that the GP is aware of.

What investigations to arrange?

A

Dependent on wells score -

<4 - D-dimer then consider US doppler
>4 - straight to doppler

If chest symptoms - consider CTPA.

Bloods - FBC, CRP, LFTs and U+Es for infection, +/- lactate if septic of ischaemic limb

X-ray if acute bony injury concern

How well did you know this?
1
Not at all
2
3
4
5
Perfectly
21
Q

What is the benefit of the wells score?

A

Comprehensive and ojective assessment on a limb with ?DVT to rule out.

Its benefit is it allows the clinician to make a decision on blood testing with D-dimer against US doppler which is more expensive and may be more difficult to arrange.

How well did you know this?
1
Not at all
2
3
4
5
Perfectly
22
Q

If Wells score is >4, what is the management

A

Doppler if can get within 4 hrs.

If cannot get doppler if 4hrs - send D-dimer and treat with treatment dose tinz/DOAC and get doppler within 24 hrs. Would refer to local guidelines for best treatment options for tinz.

How well did you know this?
1
Not at all
2
3
4
5
Perfectly
23
Q

How would you counsel someone before starting anticoagulation?

A

Patients starting anticoagulation should be given both verbal and written information on how to use the type of anticoagulation; the duration of treatment; possible side effects and how to manage these; the effects of other medications and over-the-counter medications on their effects; making them aware that they should discuss its use with medical professionals before planned procedures or becoming pregnant.

How well did you know this?
1
Not at all
2
3
4
5
Perfectly
24
What further investigations may be helpful to investigate an unprovoked DVT?
Need referral to thrombolysis clinic to look for diseases such as factor V leiden, protein C resistance etc. Need to consider investigations for cancer if clinic suspicion based on hx and ex
25
How to break bad news to miss jacobs who has ovarian cancer on CT after DVT confirmed.
SPIKES - Setting up the conversation, Perception - assessing the patient's Perception, Invitation to talk - obtaining the patient's Invitation to share information, Knowledge delivering Knowledge clearly Empathy - addressing Emotions with empathy, Strategy for follow-up care and support.
26
Who can help with BBB
Senior discussions Nurse specialists for oncology if available HCA/nurses for emotional support for patient
27
You are the SHO in Ambulatory Care. You are asked to see Mrs Jacobs, a 56-year-old lady who has been referred in by her GP with a painful left calf. You are asked to review her and organise the appropriate investigations. She has noticed her left leg has become painful and swollen over the last few days. She has no past medical history that the GP is aware of Patient has a confirmed DVT and now CT based on GI symptoms shows ovarian cancer. PT informed. Now handover patient to reg/consultant
Situation: This 56-year-old woman has presented with a left DVT. Investigations have revealed that she has an underlying ovarian cancer. Background: She has no past medical history of note. Assessment: She has been started on a DOAC for treatment of her DVT and given information on the medication and how to take it. She has been informed about the likely ovarian cancer finding and given information on what happens next. Recommendations: This lady will need an urgent referral along the two week wait pathway to the oncology team. She will need a referral to the anticoagulation clinic to review her DOAC once initiated. I think she would benefit from a discussion today with one of the senior medical team (registrar/consultant) given the new diagnosis of ovarian cancer.
28
You are the IMT 1 on the ward. The nurses call you to go and see a 45 year old man who has been having a seizure for 2 minutes. They have been admitted under the Stroke team after presenting with a sub arachnoid haemorrhage due to a ruptured ACOM aneurysm which has been coiled. Their admission has been complicated by recurrent chest infections. How would you assess this patient
Start with ABCDE Airway - check airway is patent, difficult if having seizure so try triple airway manouvure and NP if needed. Breathing - Administer 100% oxygen via a non-rebreathe mask and start continuous saturation monitoring. If saturations are dropping or there are signs of cyanosis, immediately put out a resuscitation call. Circulation - assess BP and HR, get IV access and send of bloods and VBG. IV access may be needed for IV lorazepam. Ask nurse to prep 4mg IV Disability - get BM, check neuro exam once stop seizing. Exposure - check temp and expose for rashes or injuries
29
You are the IMT 1 on the ward. The nurses call you to go and see a 45 year old man who has been having a seizure for 2 minutes. They have been admitted under the Stroke team after presenting with a sub arachnoid haemorrhage due to a ruptured ACOM aneurysm which has been coiled. Their admission has been complicated by recurrent chest infections. The patient continues having generalised tonic clonic seizures approaching five minutes. What would be your initial management of this patient?
IV lorazepam --> IV lorazepam (call ITU and senior) --> phenytoin and call ITU --> intubation
30
You are the IMT 1 on the ward. The nurses call you to go and see a 45 year old man who has been having a seizure for 2 minutes. They have been admitted under the Stroke team after presenting with a sub arachnoid haemorrhage due to a ruptured ACOM aneurysm which has been coiled. Their admission has been complicated by recurrent chest infections. What investigations to order
BMs - check for hypos FBC, LFTs, U+Es, CRP, Mg, calcium, phosphate - check for electroltye abnormalities, infection that can cause seizures or hepatic encephalopathy CT head - make sure no rebleeding/raised ICP post coil.
31
You are the IMT 1 on the ward. The nurses call you to go and see a 45 year old man who has been having a seizure for 2 minutes. They have been admitted under the Stroke team after presenting with a sub arachnoid haemorrhage due to a ruptured ACOM aneurysm which has been coiled. Their admission has been complicated by recurrent chest infections. What may be the cause of the seizure?
Most likely is increased seizure post-brain injury May be epileptic Hypos Electrolyte disturbances Rebleeding post-coil New medications Infections
32
You are the IMT 1 on the ward. The nurses call you to go and see a 45 year old man who has been having a seizure for 2 minutes. They have been admitted under the Stroke team after presenting with a sub arachnoid haemorrhage due to a ruptured ACOM aneurysm which has been coiled. Their admission has been complicated by recurrent chest infections. Managed them with two doses of benzos but not stopped seizing.The Resuscitation Team arrive to assist you in managing this patient. Please take one minute to hand over this patient to the Acute Medical Registrar.
Situation: This is a 45 year old gentleman in persistent convulsive status despite two treatments with buccal midazolam of 10mg each. Background: He has been admitted following a sub arachnoid haemorrage due to a rupture of an ACOM aneurysm which has been coiled. He is not known to have had seizures previously but has had several chest infections. Assessment: Seizures have continued for 10 minutes despite two doses of 10mg buccal midazolam. The patient is tachycardic, hypertensive and saturating at 94% on 15L of oxygen. I have requested blood tests including a VBG to assess for any electrolyte disturbance of evidence of hypoglycaemia. Recommendations: Now that the medical team have arrived to support you can consider escalation of treatment, airway management, and prioritisation of further tests to identify the cause of the seizure.
33
You are the IMT1 on call on the acute medical take. You have been asked to review a 25 year old female patient referred by their GP with an acute onset of sharp chest pain and shortness of breath. She has no significant past medical history. Her only regular medication is the combined oral contraceptive pill. She does not smoke. Her observations in A&E are as follows: Temp 37.2, RR 32, Sats 93% on air, HR 108, BP 112/65 An arterial blood gas has already been performed which demonstrates the following (taken on air, with no supplemental oxygen): pH 7.45 (7.35-7.45) , pO2 7.7 (10-14), pCO2 3.9 (4.5-6.0) Na 138 (135-145), K+ 4.5 (3.5-5.5), Glu 5.9 (4-8) HCO3 24 (22-26), BE -0.5 (-2 -> +2), Lactate 1.0 (<2.0) How would you initially assess and manage this patient?
A-E assessment Safe to approach with PPE etc Airway - patent and no compromise then move to B Breathing - RR, sats, chest expansion, trachea, auscultation, ABG, CXR, ABG shows T1 RF therefore 15litres non-rebreathe Circulation - CRT, HR, pulse, ECG as tachycardia, heart sounds, IV access to get fbc, crp, lfts, u+es, d-dimer. D - GCS, BMs, E - temp and review head to toe including calves for leg swellings. Once stable - take thorough hx
34
You are the IMT1 on call on the acute medical take. You have been asked to review a 25 year old female patient referred by their GP with an acute onset of sharp chest pain and shortness of breath. She has no significant past medical history. Her only regular medication is the combined oral contraceptive pill. She does not smoke. Her observations in A&E are as follows: Temp 37.2, RR 32, Sats 93% on air, HR 108, BP 112/65 An arterial blood gas has already been performed which demonstrates the following (taken on air, with no supplemental oxygen): pH 7.45 (7.35-7.45) , pO2 7.7 (10-14), pCO2 3.9 (4.5-6.0) Na 138 (135-145), K+ 4.5 (3.5-5.5), Glu 5.9 (4-8) HCO3 24 (22-26), BE -0.5 (-2 -> +2), Lactate 1.0 (<2.0) What is your differential diagnosis at this point?
Most likely is a PE. Especially due to T1 RF Others are - pneumothorax, pleurisy + infection, asthma, costochondritis, pericarditis
35
You are the IMT1 on call on the acute medical take. You have been asked to review a 25 year old female patient referred by their GP with an acute onset of sharp chest pain and shortness of breath. She has no significant past medical history. Her only regular medication is the combined oral contraceptive pill. She does not smoke. Her observations in A&E are as follows: Temp 37.2, RR 32, Sats 93% on air, HR 108, BP 112/65 An arterial blood gas has already been performed which demonstrates the following (taken on air, with no supplemental oxygen): pH 7.45 (7.35-7.45) , pO2 7.7 (10-14), pCO2 3.9 (4.5-6.0) Na 138 (135-145), K+ 4.5 (3.5-5.5), Glu 5.9 (4-8) HCO3 24 (22-26), BE -0.5 (-2 -> +2), Lactate 1.0 (<2.0) Are there any other investigations you would like to perform?
Wells score, likely needs CTPA and CXR first. COVID and flu swabs, bloods as suggested.
36
You are the IMT1 on call on the acute medical take. You have been asked to review a 25 year old female patient referred by their GP with an acute onset of sharp chest pain and shortness of breath. She has no significant past medical history. Her only regular medication is the combined oral contraceptive pill. She does not smoke. Her observations in A&E are as follows: Temp 37.2, RR 32, Sats 93% on air, HR 108, BP 112/65 An arterial blood gas has already been performed which demonstrates the following (taken on air, with no supplemental oxygen): pH 7.45 (7.35-7.45) , pO2 7.7 (10-14), pCO2 3.9 (4.5-6.0) Na 138 (135-145), K+ 4.5 (3.5-5.5), Glu 5.9 (4-8) HCO3 24 (22-26), BE -0.5 (-2 -> +2), Lactate 1.0 (<2.0) The patient has a chest x-ray subsequently performed which shows: “Moderate right sided pneumothorax measuring 3.5cm. Otherwise clear lung fields.” What’s the next stage of management
As hypoxic, needs chest drain so to discuss with medical SpR. Discharged once ABG improves and symptoms resolve. Important to inform regarding no flying for 7 days and banned from scubadiving Repeat CXR in 4 weeks
37
You are the IMT1 on call on the acute medical take. You have been asked to review a 25 year old female patient referred by their GP with an acute onset of sharp chest pain and shortness of breath. She has no significant past medical history. Her only regular medication is the combined oral contraceptive pill. She does not smoke. Her observations in A&E are as follows: Temp 37.2, RR 32, Sats 93% on air, HR 108, BP 112/65 An arterial blood gas has already been performed which demonstrates the following (taken on air, with no supplemental oxygen): pH 7.45 (7.35-7.45) , pO2 7.7 (10-14), pCO2 3.9 (4.5-6.0) Na 138 (135-145), K+ 4.5 (3.5-5.5), Glu 5.9 (4-8) HCO3 24 (22-26), BE -0.5 (-2 -> +2), Lactate 1.0 (<2.0) The patient has a chest x-ray subsequently performed which shows: “Moderate right sided pneumothorax measuring 3.5cm. Otherwise clear lung fields.” You have one minute to hand over the patient to your registrar/consultant as if you were in the acute medical handover.
Situation: Hi I’m ‘x’, the IMT1 on the acute take. I need to hand over a 25 year old woman patient who has presented with a primary spontaneous pneumothorax. Background: She presented with chest pain and has no significant known medical history. Assessment: I have performed an A to E assessment which revealed tachypnoea and hypoxia so I started her on 15L oxygen via a non-rebreather mask. Her ABG showed a type 1 respiratory failure and chest x-ray revealed a 3.5cm pneumothorax. Recommendation: This patient requires urgent treatment with either needle aspiration or a chest drain insertion. I would be grateful for your senior input with regards to any further advice on investigations and management.
38
You are the IMT1 on call on the acute medical take. You have been asked to review a 25 year old female patient referred by their GP with an acute onset of sharp chest pain and shortness of breath. She has no significant past medical history. Her only regular medication is the combined oral contraceptive pill. She does not smoke. Her observations in A&E are as follows: Temp 37.2, RR 32, Sats 93% on air, HR 108, BP 112/65 An arterial blood gas has already been performed which demonstrates the following (taken on air, with no supplemental oxygen): pH 7.45 (7.35-7.45) , pO2 7.7 (10-14), pCO2 3.9 (4.5-6.0) Na 138 (135-145), K+ 4.5 (3.5-5.5), Glu 5.9 (4-8) HCO3 24 (22-26), BE -0.5 (-2 -> +2), Lactate 1.0 (<2.0) The patient has a chest x-ray subsequently performed which shows: “Moderate right sided pneumothorax measuring 3.5cm. Otherwise clear lung fields.” Would you discharge pt
Would discharge if resolution of symptoms and ABG improved. Can get ambo care review of chest drain if still in situ. This is key scenario where safety netting is vital.
39
You are the IMT1 working on the acute medical take. You have been asked to assess a 44 year old male patient who has been brought in by ambulance following a suspected overdose. Several empty packets of medications were found by the ambulance crew in his home, including paracetamol. He has a background of depression but no other past medical history. His initial observations on arrival in A&E are as follows: Temperature 35.8, RR 10, Sats 95% on air, HR 80, BP 112/69 He appears drowsy on your initial assessment and smells of alcohol How would you initially assess and manage this patient?
If safe to proceed I would to A-E assessment A - check airway is patent, may need adjuncts depending how drowsy he is. If needs airway support will get urgent help. B - RR (particularly important in overdoses), sats, chest expansion, trachea position, chest auscultation (?aspiration), no ABG needed atm but may need CXR C - HR, CRT, BP, cannula with FBC, CRP, LFTs, U+Es, paracetamol levels, ethanol level, salicylate level, ammonia, ECG for changes such as prolonged QT etc, fluid balance D - GCS, BMs, review pupils ?opioids, consider CT head E - review for any traumatic injuries, temp, injection sites. Review notes and redo A-E
40
You are the IMT1 working on the acute medical take. You have been asked to assess a 44 year old male patient who has been brought in by ambulance following a suspected overdose. Several empty packets of medications were found by the ambulance crew in his home, including paracetamol. He has a background of depression but no other past medical history. His initial observations on arrival in A&E are as follows: Temperature 35.8, RR 10, Sats 95% on air, HR 80, BP 112/69 He appears drowsy on your initial assessment and smells of alcohol What investigations would you like to perform?
FBC, CRP, LFTs, U+Es, lactate, VBG, paracetamol level, salicylate level, alcohol level. ECG - complications of overdoses (prolonged QRS/QT) CXR if aspiration Review toxbase to find out best management.
41
You are the IMT1 working on the acute medical take. You have been asked to assess a 44 year old male patient who has been brought in by ambulance following a suspected overdose. Several empty packets of medications were found by the ambulance crew in his home, including paracetamol. He has a background of depression but no other past medical history. His initial observations on arrival in A&E are as follows: Temperature 35.8, RR 10, Sats 95% on air, HR 80, BP 112/69 He appears drowsy on your initial assessment and smells of alcohol. Subsequent blood tests reveal a very high paracetamol level approximately 5 hours after the medications were taken. How would you now manage this pt?
Consult local guidelines to review paracetamol level and whether above treatment line for NAC. Update NOK Consider MH team input/liason psychiatry for when patient better
42
You are the IMT1 working on the acute medical take. You have been asked to assess a 44 year old male patient who has been brought in by ambulance following a suspected overdose. Several empty packets of medications were found by the ambulance crew in his home, including paracetamol. He has a background of depression but no other past medical history. His initial observations on arrival in A&E are as follows: Temperature 35.8, RR 10, Sats 95% on air, HR 80, BP 112/69 He appears drowsy on your initial assessment and smells of alcohol. Subsequent blood tests reveal a very high paracetamol level approximately 5 hours after the medications were taken. Please outline the primary mechanisms of the toxic effect of paracetamol and how this relates to the treatment options.
Paracetamol uses up glutathione which causes NAPQI accumulation. NAC acts as a precursor for glutathione and thus stops accumulation of hepatotoxic NAPQI
43
You are the IMT1 working on the acute medical take. You have been asked to assess a 44 year old male patient who has been brought in by ambulance following a suspected overdose. Several empty packets of medications were found by the ambulance crew in his home, including paracetamol. He has a background of depression but no other past medical history. His initial observations on arrival in A&E are as follows: Temperature 35.8, RR 10, Sats 95% on air, HR 80, BP 112/69 He appears drowsy on your initial assessment and smells of alcohol. Subsequent blood tests reveal a very high paracetamol level approximately 5 hours after the medications were taken. You have one minute to please handover this patient to your registrar/consultant as if you were in the acute medical handover.
Situation: Hi I’m ‘x’, the IMT1 on the acute medical take. I need to hand over a 44 year old male patient who presented with a mixed overdose including paracetamol. Background: He has a background of depression and was found at home by paramedics with several packets of medication. Assessment: I have performed an A-E assessment upon which he appeared to be haemodynamically stable and taken initial blood tests which have shown a significantly raised paracetamol level. Recommendation: I believe this patient requires urgent treatment with a NAC infusion and further supportive measures as needed. I would be grateful for your senior input with regards to ongoing investigations and management.
44
You are the IMT1 working on the acute medical take. You have been asked to assess a 44 year old male patient who has been brought in by ambulance following a suspected overdose. Several empty packets of medications were found by the ambulance crew in his home, including paracetamol. He has a background of depression but no other past medical history. His initial observations on arrival in A&E are as follows: Temperature 35.8, RR 10, Sats 95% on air, HR 80, BP 112/69 He appears drowsy on your initial assessment and smells of alcohol. Subsequent blood tests reveal a very high paracetamol level approximately 5 hours after the medications were taken. Over the next 48 hours, the patient’s U&Es, LFTs and coagulation profile remain grossly abnormal despite your treatment. Are there any other management options to be considered?
Will depend on number of factors such as creatinine, ALT, PT, pH and encephalopathy. Based on these, may require liver transplant, therefore discussion with local liver transplant centre will be required
45
Can you describe any classification systems for ischaemic stroke?
The Bamford classification splits ischaemic strokes into the following: Total Anterior Circulation Stroke (TACS) - 3/3 features of weakness, hemianopia, cognitive Partial Anterior Circulation Stroke (PACS) - ⅔ features or higher cognitive dysfunction alone Posterior Circulation Syndrome (POCS) - posterior/cerebellar signs Lacunar Syndrome (LACS) - pure motor/sensory without higher cognitive dysfunction Important in prognosis
46
What would you like to do to control the blood pressure in this stroke case?
“Anti-hypertensive treatment in people with acute ischaemic stroke is recommended only if there is a hypertensive emergency” “Blood pressure reduction to 185/110 mmHg or lower should be considered in people who are candidates for intravenous thrombolysis” If haemorrhagic stroke - GTN/IV labetalol required
47
You are the IMT1 on call on the acute medical take. You have been asked to see a 65-year-old male patient who has presented with a 1 day history of severe left knee pain and swelling. He has a background of atrial fibrillation, type 2 diabetes mellitus and hypertension. His observations are as follows: Temp 38.0, RR 20, sats 98% on air, HR 110, BP 116/72 How would you initially assess and manage this patient?
Make sure safe to approach ABCDE A - make sure patent and talking, if not seek urgent medical advice and check for signs of life. B - RR, sats, chest expansion, trachea, chest auscultation, CXR due to high RR and temp, Consider ABG if desaturates C - HR, CRT, BP, ECG as tachycardic, auscultate chest, Cannula with FBC, CRP, LFTs, lactate, BC, U+Es, ?d-dimer, check for overload and JVP, give abx and IVT D - GCS, BMs and check pupils E - look for rashes, cellulitis, examine other joints and joints above and below. Examine Knee for erythema, ROM etc Start sepsis 6 Once complete - take a full Hx
48
You are the IMT1 on call on the acute medical take. You have been asked to see a 65-year-old male patient who has presented with a 1 day history of severe left knee pain and swelling. He has a background of atrial fibrillation, type 2 diabetes mellitus and hypertension. His observations are as follows: Temp 38.0, RR 20, sats 98% on air, HR 110, BP 116/72 What is your differential diagnosis at this point?
Septic arthritis Cellulitis Gout/pseudogout Reactive arthritis Trauma
49
You are the IMT1 on call on the acute medical take. You have been asked to see a 65-year-old male patient who has presented with a 1 day history of severe left knee pain and swelling. He has a background of atrial fibrillation, type 2 diabetes mellitus and hypertension. His observations are as follows: Temp 38.0, RR 20, sats 98% on air, HR 110, BP 116/72 What would be your initial plan for investigation and management?
Sepsis 6 X-ray of knee Discuss with ortho regarding joint aspiration
50
You are the IMT1 on call on the acute medical take. You have been asked to see a 65-year-old male patient who has presented with a 1 day history of severe left knee pain and swelling. He has a background of atrial fibrillation, type 2 diabetes mellitus and hypertension. His observations are as follows: Temp 38.0, RR 20, sats 98% on air, HR 110, BP 116/72 The patient subsequently has a joint aspiration performed, which shows: “Thick, turbid, blood-stained fluid. Presence of gram positive cocci” How will this affect your management plan? Are there any other teams or specialties you would like to involve?
Likely staph or strep. Indicates septic joint - give high dose 2g fluclox QDS for penetration and discuss with ortho regarding need for washout. If became more unwell - would need discussion with critical care
51
What would you expect to find on joint aspirates for gout / pseudogout?
Gout - negatively birefringent needle-shaped crystals. Urate. Pseudogout - positively birefringent rhomboid-shaped crystals. calcium pyrophosphate
52
Are you able to name any risk factors for septic arthritis?
Previous septic arthritis, prosthetic joint, local penetrating infection, immunosuppression, STI (Gonorrhoea)
53
You are the IMT1 on call on the acute medical take. You have been asked to see a 65-year-old male patient who has presented with a 1 day history of severe left knee pain and swelling. He has a background of atrial fibrillation, type 2 diabetes mellitus and hypertension. His observations are as follows: Temp 38.0, RR 20, sats 98% on air, HR 110, BP 116/72 The patient subsequently has a joint aspiration performed, which shows: “Thick, turbid, blood-stained fluid. Presence of gram positive cocci” You have one minute to hand over the patient to your registrar/consultant as if you were in the acute medical handover.
Situation: Hello, I’m ‘x’, the IMT1 on the acute medical take. I need to hand over a 65 year old male who has presented with joint pain and fever suspicious for possible septic arthritis. Background: He has a background of AF, T2DM and hypertension. He presented today with a 1 day history of left knee pain. Assessment: I have performed an A-E assessment in which he was febrile and had left knee joint swelling. I have started treatment with IV antibiotics and fluid, have sent blood tests including cultures, and we are performing a joint aspiration. Recommendation: I would greatly appreciate senior input for this patient in order to guide further investigation & management, which I think is likely to involve admission and orthopaedic input for ongoing IV antibiotics and joint aspirations/washout.
54
You are the IMT 1 on the acute medical take. You have been asked to see a 27 year old female patient admitted with a 1 day history of severe right sided flank pain and vomiting. She has recently been treated for a urinary tract infection by her GP. She has no known past medical history and does not currently take any medications. Her observations on arrival are as follows: Temp 38.2, RR 20, sats 98% on air, HR 116, BP 94/60 How would you initially assess and manage this patient?
Check for danger by assessing trust guidelines if PPE is required. ABCDE A - I would assess whether the patient is able to maintain their own airway. If they are alert with no signs of airway compromise I would move on to assess B. B - Review sats, RR, Chest expansion, auscultation and trachea positioning. Would then consider infection screen and get CXR. No need for ABG at this point. May need covid/flu swab. C - HR, CRT, Pulse rate and rhythm, ECG as tachycardia, Listen to heart, assess JVP and for peripheral oedema, get IV access for FBC, CRP, LFTs, U+Es, urine dip and PREGNANCY TEST, amylase, BC and lactate. Give IVT and abx D - check GCS, BMs E - look for rashes and assess abdomen for peritonism and flanks for pain. Ensure not in urinary retention, check temp Once patient is stable and A-E complete. I will review her notes and reassess pt
55
You are the IMT 1 on the acute medical take. You have been asked to see a 27 year old female patient admitted with a 1 day history of severe right sided flank pain and vomiting. She has recently been treated for a urinary tract infection by her GP. She has no known past medical history and does not currently take any medications. Her observations on arrival are as follows: Temp 38.2, RR 20, sats 98% on air, HR 116, BP 94/60 What would be your differential diagnosis for this patient?
Pyelonephritis - top differential Renal stone Psoas abscess Acute cholecystitis Ruptured ectopic or septic miscarriage CAP Pancreatitis
56
You are the IMT 1 on the acute medical take. You have been asked to see a 27 year old female patient admitted with a 1 day history of severe right sided flank pain and vomiting. She has recently been treated for a urinary tract infection by her GP. She has no known past medical history and does not currently take any medications. Her observations on arrival are as follows: Temp 38.2, RR 20, sats 98% on air, HR 116, BP 94/60 What would be your initial investigations and management plan?
Bedside: I would perform an ECG, check a pregnancy test, and also send a urine sample for microscopy & culture. I would also send COVID swabs. Imaging - CXR, US of pelvis if worried about ectopic/retained tissues. May need CT if concerned about obstruction once discuss with surgeons.
57
You are the IMT 1 on the acute medical take. You have been asked to see a 27 year old female patient admitted with a 1 day history of severe right sided flank pain and vomiting. She has recently been treated for a urinary tract infection by her GP. She has no known past medical history and does not currently take any medications. Her observations on arrival are as follows: Temp 38.2, RR 20, sats 98% on air, HR 116, BP 94/60 Her VBG is performed and shows the following key results: pH 7.35 (7.35-7.45) HCO3 24 (22-26) Glucose 5.5 (4-8) Lac 2.9 (<2.0) How does this affect your management plan?
High lactate - suggestive of poor tissue perfusion and or catecholamine response to infection and concern for sepsis. Early management with fluid bolus(es) and antibiotics is therefore crucial along with completing a sepsis 6 bundle.
58
You are the IMT 1 on the acute medical take. You have been asked to see a 27 year old female patient admitted with a 1 day history of severe right sided flank pain and vomiting. She has recently been treated for a urinary tract infection by her GP. She has no known past medical history and does not currently take any medications. Her observations on arrival are as follows: Temp 38.2, RR 20, sats 98% on air, HR 116, BP 94/60 Her VBG is performed and shows the following key results: pH 7.35 (7.35-7.45) HCO3 24 (22-26) Glucose 5.5 (4-8) Lac 2.9 (<2.0) 24 hours later, the microbiology team call you to report that her urine cultures have grown an Extended Spectrum Beta-Lactamase (ESBL)-producing E. Coli. The pregnancy test is negative. How does this affect your management plan?
Likely pyelonephritis with resisitant organism due to recent failed abx from GP. Need to discuss with micro
59
What do you understand by the term ‘antimicrobial stewardship’ and the importance of antimicrobial selection for global public health?
Increasing antimicrobial resistance worldwide as ABx become more widely accessibly in healthcare and farming. Inappropriate antibiotic prescribing is a major contributor to the problem. Antimicrobial stewardship and ensuring judicious prescribing practices for antibiotics is therefore vital for worldwide health systems as part of a range of measures to tackle the issue.
60
You are the IMT 1 on the acute medical take. You have been asked to see a 27 year old female patient admitted with a 1 day history of severe right sided flank pain and vomiting. She has recently been treated for a urinary tract infection by her GP. She has no known past medical history and does not currently take any medications. Her observations on arrival are as follows: Temp 38.2, RR 20, sats 98% on air, HR 116, BP 94/60 Her VBG is performed and shows the following key results: pH 7.35 (7.35-7.45) HCO3 24 (22-26) Glucose 5.5 (4-8) Lac 2.9 (<2.0) 24 hours later, the microbiology team call you to report that her urine cultures have grown an Extended Spectrum Beta-Lactamase (ESBL)-producing E. Coli. The pregnancy test is negative. You have one minute to hand over the patient to your registrar/consultant as if you were in the acute medical handover.
Situation: Hello, I’m ‘x’, the IMT1 on the acute medical take. I need to hand over a 27 year old female who has presented with suspected pyelonephritis. Background: She presented with a 1 day history of flank pain and vomiting following recent treatment for a urinary tract infection by her GP. Assessment: I have performed an A-E assessment in which she was found to be febrile, tachycardic and hypotensive. I have initiated treatment with IV fluids and broad spectrum antibiotics. Her VBG revealed a raised lactate and urine cultures have demonstrated a resistant E.coli organism. Recommendation: I would greatly appreciate senior input for this patient in order to guide further investigation & management for suspected pyelonephritis. I plan to consult the microbiology team to ensure appropriate antimicrobial coverage and consider any imaging.
61
You are the IMT1 covering the acute medical unit. The nurse has called you to come and review James, a 19-year-old male patient who has been admitted with an exacerbation of their asthma. They were seen in A+E, transferred straight to the ward, and have not yet been seen by the medical team. You have been asked to see them. The nurse tells you they are having some difficulty breathing. They have had asthma as a childhood, but have never before been hospitalised due to asthma, and have no other significant past medical history. What other diagnoses would you consider on your way to seeing this patient?
COVID/Flu CAP PE Anaphylaxis Pneumothorax
62
You are the IMT1 covering the acute medical unit. The nurse has called you to come and review James, a 19-year-old male patient who has been admitted with an exacerbation of their asthma. They were seen in A+E, transferred straight to the ward, and have not yet been seen by the medical team. You have been asked to see them. The nurse tells you they are having some difficulty breathing. They have had asthma as a childhood, but have never before been hospitalised due to asthma, and have no other significant past medical history. What do you need to consider when assessing someone admitted with an acute asthma attack?
Low RR (fatiging) O2 and CO2 - tiring? Peak flow If talking in full sentences or not GCS Sats <92%
63
You are the IMT1 covering the acute medical unit. The nurse has called you to come and review James, a 19-year-old male patient who has been admitted with an exacerbation of their asthma. They were seen in A+E, transferred straight to the ward, and have not yet been seen by the medical team. You have been asked to see them. The nurse tells you they are having some difficulty breathing. They have had asthma as a childhood, but have never before been hospitalised due to asthma, and have no other significant past medical history. How would you assess a patient with an acute asthma attack and what bedside tests would you perform to help determine the asthma attack's severity?
Put on PPE if required. ABCDE A - I would assess the airway for any signs of obstruction, listening for sounds such as stridor or wheeze. B - RR, if they can speak in full sentences, sats, chest auscultation - ?wheeze, percussion, expansion, trachea position, ABG, CXR, peak flow in order to grade severity, give salbutamol and ipatropium nebs, Covid and flu swab C - HR, CRT, BP, Cannula, FBC, CRP, LFTs, U+Es, BC, consider IV Mg with senior input, D-dimer if needed. D - GCS and BMs E - temp and check for rashes.
64
You are the IMT1 covering the acute medical unit. The nurse has called you to come and review James, a 19-year-old male patient who has been admitted with an exacerbation of their asthma. They were seen in A+E, transferred straight to the ward, and have not yet been seen by the medical team. You have been asked to see them. The nurse tells you they are having some difficulty breathing. They have had asthma as a childhood, but have never before been hospitalised due to asthma, and have no other significant past medical history. This patient has the following observations: Saturations 91% on room air, respiratory rate 26, heart rate 127 bpm, temperature 36.9 and a blood pressure of 110/78. In addition, his peak flow is 30% of predicted. How would you manage a life-threatening asthma attack?
ABCDE Give oxygen, salbutamol, ipatropium, hydrocortisone Needs urgent ITU and med reg input to discuss with crit care for consideration of IV magnesium and NIV.
65
You are the IMT1 covering the acute medical unit. The nurse has called you to come and review James, a 19-year-old male patient who has been admitted with an exacerbation of their asthma. They were seen in A+E, transferred straight to the ward, and have not yet been seen by the medical team. You have been asked to see them. The nurse tells you they are having some difficulty breathing. They have had asthma as a childhood, but have never before been hospitalised due to asthma, and have no other significant past medical history. This patient has the following observations: Saturations 91% on room air, respiratory rate 26, heart rate 127 bpm, temperature 36.9 and a blood pressure of 110/78. In addition, his peak flow is 30% of predicted. Do you know any criteria that would prompt you to discuss this patient with ITU?
If deteriorating PEF, rising CO2, dropping consciousness or RR Need to discuss for NIV
66
You are the IMT1 covering the acute medical unit. The nurse has called you to come and review James, a 19-year-old male patient who has been admitted with an exacerbation of their asthma. They were seen in A+E, transferred straight to the ward, and have not yet been seen by the medical team. You have been asked to see them. The nurse tells you they are having some difficulty breathing. They have had asthma as a childhood, but have never before been hospitalised due to asthma, and have no other significant past medical history. This patient has the following observations: Saturations 91% on room air, respiratory rate 26, heart rate 127 bpm, temperature 36.9 and a blood pressure of 110/78. In addition, his peak flow is 30% of predicted. You have started treatment with the patient, but they collapse whilst you are on the ward. What would you do?
I would immediately put out a ‘2222’ adult cardiac arrest call and call for help in the ward. I would start my assessment as per the ALS guidelines and start CPR. High flow O2 and tidal flow of 8-10 RR Likely need intubation with anaesthetic team input Go through Ts and Hs
67
Name the reversible causes of a cardiac arrest
4 “Ts” and 4 “Hs” causes - (hypoxia, hypokalaemia/hyperkalaemia, hypothermia/hyperthermia, hypovolaemia) (tension pneumothorax, tamponade, thrombosis, toxins)
68
You are the IMT1 covering the acute medical unit. The nurse has called you to come and review James, a 19-year-old male patient who has been admitted with an exacerbation of their asthma. They were seen in A+E, transferred straight to the ward, and have not yet been seen by the medical team. You have been asked to see them. The nurse tells you they are having some difficulty breathing. They have had asthma as a childhood, but have never before been hospitalised due to asthma, and have no other significant past medical history. This patient has the following observations: Saturations 91% on room air, respiratory rate 26, heart rate 127 bpm, temperature 36.9 and a blood pressure of 110/78. In addition, his peak flow is 30% of predicted. The patient the collpases and has cardiac arrest due to hypoxia The patient is successfully resuscitated and taken to ITU. The patient's sister has arrived on the ward and your consultant has asked you to talk to them about what has happened. What should you consider before talking to the patient’s relative?
Do spikes model - find quiet room, find out what she knows, deliver bad news. Offer support and get her access to ITU once I liaise with them.
69
You are the IMT1 covering the acute medical unit. The nurse has called you to come and review James, a 19-year-old male patient who has been admitted with an exacerbation of their asthma. They were seen in A+E, transferred straight to the ward, and have not yet been seen by the medical team. You have been asked to see them. The nurse tells you they are having some difficulty breathing. They have had asthma as a childhood, but have never before been hospitalised due to asthma, and have no other significant past medical history. This patient has the following observations: Saturations 91% on room air, respiratory rate 26, heart rate 127 bpm, temperature 36.9 and a blood pressure of 110/78. In addition, his peak flow is 30% of predicted. The patient is discussed at the medical handover at the end of your shift. Please sum up the course of events and hand over to the medical team.
S: This is a 19-year-old who is currently receiving post-resus care on ITU, having had a cardiac arrest due to a life-threatening asthma attack. B: He has a background of childhood asthma but has never previously been admitted to hospital due to an exacerbation. A: On initial assessment, he was found to be having a life-threatening asthma attack due to hypoxia, and a PEF of 30% of predicted. He was initially treated with steroids, β₂ agonists, ipratropium nebs and IV magnesium sulfate. However, he subsequently arrested on the ward, and was resuscitated following a 2222 call. He is currently intubated on ITU. I have spoken with the patient's relatives about what has happened, after discussion with the senior medical team. R: he will need to be re-assessed on ITU, and further investigations such as a chest x-ray arranged. I would suggest that a follow up consultation is held with the family following the post-take ward round as they are likely to have further questions and will need to be updated with the patient's progress. Once stabilised, he will need to be reviewed to establish the cause of the initial exacerbation, and I would suggest that they are referred to the respiratory team to be reviewed during this admission and assess the management of their asthma.
70
You are the IMT 2 assisting with a Falls Clinic in a community geriatric unit. You have been asked to see Mrs Redbridge, an 87-year-old woman who was referred by her GP following repeated falls at home. She has a past medical history of coronary vascular disease, hypertension, and type two diabetes. What would be the key aspects of the history that would help you determine the likely cause of her falls?
First start with open question asking patient to describe what happened. Would then go into more closed questions - Proceeding symptoms- lightheadedness/dizziness/CP What she was doing leading up to fall If lost conciousness Any inuries Symptoms after the fall - confusion, visual changes, weakness How long she was on floor Neuro exam to ensure no peripheral neuropathy If this is presenting like usual falls or different. Then review medication and PMH Then review SH - who she lives with and their health, walking aids, home layout, alcohol/drugs Brief systematic review
71
You are the IMT 2 assisting with a Falls Clinic in a community geriatric unit. You have been asked to see Mrs Redbridge, an 87-year-old woman who was referred by her GP following repeated falls at home. She has a past medical history of coronary vascular disease, hypertension, and type two diabetes. What investigations would be helpful to aid your diagnosis?
Obs, L/S BP Bloods - CRP, LFTs, U+Es, FBC to assess for infection and anaemia X-ray of any joints/bony pain - ?OA contributing to fall. May need holter monitor if cardiac syncope concerns. Dix-Hallpike test if vertigo
72
You are the IMT 2 assisting with a Falls Clinic in a community geriatric unit. You have been asked to see Mrs Redbridge, an 87-year-old woman who was referred by her GP following repeated falls at home. She has a past medical history of coronary vascular disease, hypertension, and type two diabetes. Would you order a CT head?
This would be based on my history, examination and investigation conducted in clinic. If there is a history of head injury associated with loss of consciousness or change in cognitive function, I would consider a CT head. If the patient is on any anticoagulant medication this would also change my thinking. I would discuss this request with my senior colleagues before ordering it.
73
You are the IMT 2 assisting with a Falls Clinic in a community geriatric unit. You have been asked to see Mrs Redbridge, an 87-year-old woman who was referred by her GP following repeated falls at home. She has a past medical history of coronary vascular disease, hypertension, and type two diabetes. The CT Head report shows a chronic subdural haematoma. You have discussed this with the on-call neurosurgical team who have advised that this should be managed conservatively. She needs to be admitted for a period of inpatient monitoring. The consultant in charge of the clinic has asked you to discuss the result with the patient and her daughter. (One of the interviewers will take on the role of her daughter). Please explain the result and the plan to admit her.
SPIKES model I would start by explaining that the CT head scan has shown a cause for the worsening confusion. (This is the warning shot). I would then explain that the scan has shown a bleed around the outside of her brain. Explain that the blood is likely exerting some pressure on the brain which is why Mrs Redbridge is more confused. The bleeding is likely secondary to her use of rivaroxaban which increases her risk of haematoma formation after head injury. I would explain that the scans have been reviewed by my surgical colleagues who do not think that surgery is required. Explain that Mrs Redbridge will be admitted for a short period to monitor and assess the impact of the bleed before being discharged home. I would then make sure that the patient and family had time to ask questions. To stop DOAC temporarily.
74
You are the IMT 2 assisting with a Falls Clinic in a community geriatric unit. You have been asked to see Mrs Redbridge, an 87-year-old woman who was referred by her GP following repeated falls at home. She has a past medical history of coronary vascular disease, hypertension, and type two diabetes. The CT Head report shows a chronic subdural haematoma. You have discussed this with the on-call neurosurgical team who have advised that this should be managed conservatively. She needs to be admitted for a period of inpatient monitoring. You will now be given one minute to present this patient to your consultant as if it were the end of the clinic. Give an overview of your findings and tell us what the next steps of your management would be.
S: I have just assessed Mrs Redbridge, an 87-year-old woman referred to this clinic by her GP following repeated falls at home and new confusion. B: She has a background of coronary vascular disease, atrial fibrillation (on a DOAC), hypertension and type two diabetes. A: Based on my assessment and initial investigations, Mrs Redbridge has suffered a subdural haematoma. Based on the findings of the CT scan this appears to be chronic. I have discussed the findings with the on-call neurosurgical team. They have suggested that this is managed conservatively but that she is admitted for monitoring as an inpatient. R: I will discuss the case with the on-call medical registrar to get her admitted. I would recommend stopping the rivaroxaban as this is likely a significant contributing factor to her bleed. I would like to present the case in the neuro-vascular MDT so that her continued use of this medication can be discussed and a plan formulated. This will need to be discussed with the family as the balance of the risks and benefits of continuing or stopping this medication long term will need to be considered, given she will be at risk of stroke should we stop it. I would recommend that she is seen by the acute medical therapy team to assess her ongoing falls risks and put a plan in place to help her once she is at home.
75
You are the IMT 1 in the TIA Assessment Clinic. You have been asked to see Mr Adene, a 69-year-old gentleman who presented to A+E 24 hours earlier following a short episode of right hemiparesis and word-finding difficulty which had resolved on his admission to A+E. He was diagnosed as having a TIA and referred to your clinic for assessment. He has a background of hypertension, type two diabetes, and COPD. You will be required to assess the patient and will be asked to discuss treatment options with the patient. What questions are important to ask in the history when assessing this patient?
Start by asking about event with open questions. What happened When it happened How long symptoms lasted What symptoms occurred Any symptoms still present Did they loose consciousness Has COPD, check if still smokes. Find out social situation including home set up and alcohol hx. Important to know if patient drives for DVLA ICE
76
You are the IMT 1 in the TIA Assessment Clinic. You have been asked to see Mr Adene, a 69-year-old gentleman who presented to A+E 24 hours earlier following a short episode of right hemiparesis and word-finding difficulty which had resolved on his admission to A+E. He was diagnosed as having a TIA and referred to your clinic for assessment. He has a background of hypertension, type two diabetes, and COPD. You will be required to assess the patient and will be asked to discuss treatment options with the patient. What investigations would you order as part of your workup?
ECG (AF) Routine bloods including lipids and hba1c Carotid endarterectomy/ US MRI head
77
Differentials for TIA presentation?
Stroke Hemiplegic migraine Seizure Drug complications Wernickes encephalopathy Bells palsy Space occupying lesion
78
You are the IMT 1 in the TIA Assessment Clinic. You have been asked to see Mr Adene, a 69-year-old gentleman who presented to A+E 24 hours earlier following a short episode of right hemiparesis and word-finding difficulty which had resolved on his admission to A+E. He was diagnosed as having a TIA and referred to your clinic for assessment. He has a background of hypertension, type two diabetes, and COPD. You confirm a TIA based on your history and investigations. What advice would you give to the patient and what treatments would you start in clinic?
Explain TIA is like ministroke and puts them at high risk for a stroke so must investigate and treat risk factors to limit risk of stroke as much as possible. Lifestyle changes - stop smoking, less alcohol, low salt Dual antiplatelet therapy is recommended of clopidogrel and aspirin for 21 days, followed by monotherapy with clopidogrel. atorvastatin 80mg and antihypertensives. To discuss and check with supervisor in clinic
79
You are the IMT 1 in the TIA Assessment Clinic. You have been asked to see Mr Adene, a 69-year-old gentleman who presented to A+E 24 hours earlier following a short episode of right hemiparesis and word-finding difficulty which had resolved on his admission to A+E. He was diagnosed as having a TIA and referred to your clinic for assessment. He has a background of hypertension, type two diabetes, and COPD. Your investigations reveal a diagnosis of atrial fibrillation. Would you start the patient on treatment?
Can start treatment once confirmed no haemorrhagic factors. DOAC lifelong and rate control. To discuss and check with supervisor in clinic
80
TIA rules with driving
People who have had a TIA must not drive for one month, but they do not need to inform the DVLA. If the patient has a further TIA, then they should stop driving for 3 months and inform the DVLA at this point.
81
You are the IMT 1 in the TIA Assessment Clinic. You have been asked to see Mr Adene, a 69-year-old gentleman who presented to A+E 24 hours earlier following a short episode of right hemiparesis and word-finding difficulty which had resolved on his admission to A+E. He was diagnosed as having a TIA and referred to your clinic for assessment. He has a background of hypertension, type two diabetes, and COPD. Patient has AF and agrees to start DOAC and stop driving for 1 month You will now be given one minute to present this patient to your consultant as if it were the end of the clinic. Give an overview of your findings and tell us what the next steps of your management would be.
S: I have just assessed Mr Adene, a 69-year-old gentleman, who was referred to clinic following a transient episode of right hemiparesis and word finding difficulty. B: He has a background of hypertension, type two diabetes, and COPD. A: Based on my assessment I believe that the presentation is consistent with a transient Ischaemic Attack. I have arranged several investigations to help identify the source of the TIA and based on an ECG in clinic we have also confirmed a diagnosis of atrial fibrillation. I have spoken with Mr Adene about prevention of future stroke including lifestyle measures and have started him on high dose statin. Because of the atrial fibrillation I believe that Mr Adene would benefit from being on long term anticoagulation and have discussed this with the patient. I have spoken with him about driving and when he can return to this. R: Mr Adene should be followed up in this clinic to check his response to the medications initiated today and to review the outcome of further investigations that I have arranged including his prolonged ECG monitoring and his MRI head so that a DOAC can be started safely. I have made sure to safety net and informed Mr Adene to present back to hospital should he have any recurrence of his presenting symptoms.
82
You are the IMT 2 in a general respiratory clinic. You have been asked to assess Mrs Bruth, a 65-year-old woman who has been referred by her GP with a productive cough, wheeze and shortness of breath on exertion. She has been smoking for the last 30 years, at least 10-20 cigarettes each day. Her past medical history includes hypertension which she takes amlodipine for. She has noticed 2-3 kg of weight loss over the last 12 months. You have been asked to take a history from her and consider what investigations would help with regards to her diagnosis. What questions would be helpful for you to ask when considering her presenting complaint of cough, wheeze and shortness of breath?
Open questions - Duration of symptoms, progression of symptoms, associated symptoms - fever, malaise, exercise tolerance Cough: Is the cough productive or dry? What colour is the sputum? Is the cough persistent or intermittent? Any time of day? Any history of haemoptysis? Any chest pain when coughing? Wheeze? Does the wheeze occur spontaneously or with triggers (exercise, dust, smoke). What does the wheeze sound like? How long does the wheeze last? Has it ever been managed with medications? Dyspnoea: Does the shortness of breath occur at rest or with exertion? Is there any positional dyspnoea? How long do the episodes last and how does the patient manage it?
83
You are the IMT 2 in a general respiratory clinic. You have been asked to assess Mrs Bruth, a 65-year-old woman who has been referred by her GP with a productive cough, wheeze and shortness of breath on exertion. She has been smoking for the last 30 years, at least 10-20 cigarettes each day. Her past medical history includes hypertension which she takes amlodipine for. She has noticed 2-3 kg of weight loss over the last 12 months. You have been asked to take a history from her and consider what investigations would help with regards to her diagnosis. Is there anything else from her history you would want to know?
?Weight loss PMH and medications Social situation - who she lives with, mobility, where bathroom is, how she mobilises, occupation for exposure to asbestos etc, smoking FH of resp conditions - sarcoid, cancers etc
84
You are the IMT 2 in a general respiratory clinic. You have been asked to assess Mrs Bruth, a 65-year-old woman who has been referred by her GP with a productive cough, wheeze and shortness of breath on exertion. She has been smoking for the last 30 years, at least 10-20 cigarettes each day. Her past medical history includes hypertension which she takes amlodipine for. She has noticed 2-3 kg of weight loss over the last 12 months. You have been asked to take a history from her and consider what investigations would help with regards to her diagnosis. What investigations would be helpful in making a diagnosis?
CXR, FBC, CRP, LFTs, U+Es, Sputum culture, high resolution CT, Spirometry
85
You are the IMT 2 in a general respiratory clinic. You have been asked to assess Mrs Bruth, a 65-year-old woman who has been referred by her GP with a productive cough, wheeze and shortness of breath on exertion. She has been smoking for the last 30 years, at least 10-20 cigarettes each day. Her past medical history includes hypertension which she takes amlodipine for. She has noticed 2-3 kg of weight loss over the last 12 months. You have been asked to take a history from her and consider what investigations would help with regards to her diagnosis. What is your differential diagnosis?
Top COPD Asthma, bronchiectasis, lung cancer, TB, CHF, GORD, ACEI
86
You are the IMT 2 in a general respiratory clinic. You have been asked to assess Mrs Bruth, a 65-year-old woman who has been referred by her GP with a productive cough, wheeze and shortness of breath on exertion. She has been smoking for the last 30 years, at least 10-20 cigarettes each day. Her past medical history includes hypertension which she takes amlodipine for. She has noticed 2-3 kg of weight loss over the last 12 months. You have been asked to take a history from her and consider what investigations would help with regards to her diagnosis. Investigations come back and show COPD. You will now be given one minute to present this patient to your consultant as if it were the end of the clinic. Give an overview of your findings and tell us what the next steps of your management would be.
S: I have just assessed Mrs Bruth, a 65-year-old woman who presents with a chronic cough, wheeze and shortness of breath. B: She has a 30-pack year history and her past medical history is significant for hypertension. Of note, she has also lost about 2-3 kg over the last year. A: Based on my assessment, I think this is a presentation of chronic obstructive pulmonary disorder. I have arranged several straightforward tests to support this diagnosis, including blood tests and an X Ray. R: Because of the wide differential here, however, I would want to arrange suitable follow up for her again in this clinic to review these tests. If there is any diagnostic uncertainty following my initial investigations, I would arrange a CT Chest given the symptoms and history of weight loss. She would benefit from being seen by the Respiratory CNS to discuss treatment options and review inhalers and I will refer her to this service today. I have provided her with written information about COPD and explained to her about the importance of managing exacerbations by seeing her GP.
87
You are the on call SHO for medicine. A nurse asks you to see a 65-year-old male patient who has come in feeling drowsy and with decreased urine output. She hands you the ECG saying another doctor mentioned “tented T waves”. How would you assess this patient?
ABCDE A - does the patient maintain their airway independently? Or are they significantly drowsy such that the airway is compromised? Would use triple airway manouvure and then trial oropharyngeal. If tolerates need to shout for help quickly. B - RR, sats, chest expansion and auscultation, look for signs of resp distress and poor resp effort as drowsy C - CRT, HR, BP, ECG, FBC, CRP LFTs, U+Es, VBG for quick K and ?metabolic acidosis, 2 cannulas, Mg, and lactate. D - Drowsy so check GCS, BMs and pupils E - temp and assess for rashes abdo exam
88
You are the on call SHO for medicine. A nurse asks you to see a 65-year-old male patient who has come in feeling drowsy and with decreased urine output. She hands you the ECG saying another doctor mentioned “tented T waves”. The potassium is 7.0 on the VBG. How would you manage this patient?
Calcium gluconate (stabilise cardiac myocytes) Salbutamol nebs Insulin/dextrose infusion Cardiac monitoring Look for a cause… (renal failure, exogenous potassium, spironolactone) Calcium resonium not required for immediate management
89
You are the on call SHO for medicine. A nurse asks you to see a 65-year-old male patient who has come in feeling drowsy and with decreased urine output. She hands you the ECG saying another doctor mentioned “tented T waves”. The serum potassium comes back as 6.8. The U&Es show an AKI stage 3; the patient had normal renal function on previous blood tests 6 months ago. What are the possible causes of this AKI?
Split into pre-renal, renal and post-renal Pre-renal: volume depletion (most common), heart failure, liver failure, low BP Renal: RTA, trauma, contrast induced nephropathy, glomerulonephritis Post-renal: stones, BPH, malignancy, catheter problems
90
You are the on call SHO for medicine. A nurse asks you to see a 65-year-old male patient who has come in feeling drowsy and with decreased urine output. She hands you the ECG saying another doctor mentioned “tented T waves”. The serum potassium comes back as 6.8. The U&Es show an AKI stage 3; the patient had normal renal function on previous blood tests 6 months ago. The serum potassium comes back as 6.8. The U&Es show an AKI stage 3; the patient had normal renal function on previous blood tests 6 months ago. You commence treatment and repeat the potassium after 2 hours. The VBG result is 7.1 and there is now a metabolic acidosis. How do you proceed?
REFRACTORY HYPERKALAEMIA - Reassess using A-E. Commence another round of treatment and discuss with senior (Med reg) and crit care as may need to be on their radar for dialysis - call ITU reg.
91
What are the indications for dialysis?
AEIOU Metabolic acidosis Electrolytes - refractory hyperkalaemia Intoxicated with toxins Oedema Ureamia
92
You are the on call SHO for medicine. A nurse asks you to see a 65-year-old male patient who has come in feeling drowsy and with decreased urine output. She hands you the ECG saying another doctor mentioned “tented T waves”. The serum potassium comes back as 6.8. The U&Es show an AKI stage 3; the patient had normal renal function on previous blood tests 6 months ago. The serum potassium comes back as 6.8. The U&Es show an AKI stage 3; the patient had normal renal function on previous blood tests 6 months ago. You commence treatment and repeat the potassium after 2 hours. The VBG result is 7.1 and there is now a metabolic acidosis. You have informed med reg and ITU SpR or refractory hyperkalaemia. You now have one minute to handover the patient in this scenario to your registrar/consultant as if you were at the Acute Medical Handover.
Situation: This is a 65 year old gentleman with refractory hyperkalaemia and metabolic acidosis. Background: He was admitted in AKI and has no other known past medical history. Assessment: The potassium is 7.1 and there are changes on the ECG consistent with hyperkalaemia. The patient is drowsy. Recommendations: I would like an urgent review of this patient by the ITU registrar as he may need emergency dialysis given the lack of response to treatment so far. I will commence cardiac monitoring and continue treatment for the hyperkalaemia pending your review.
93
You are bleeped to see an 82 year old lady with significant PR bleeding in A&E. She usually lives in a nursing home and presents with a three day history of bloody diarrhoea. Her only past medical history is atrial fibrillation. Her observations are: Respiratory rate 25 breaths per minute Oxygen saturations 93% on air Blood pressure 110/65 Heart Rate 100 bpm Temperature 37.5 degrees Celsius How would you assess this patient?
Check for danger and put on PPE ABCDE A - if the patient is alert and able to talk I would consider her airway patent B - RR, sats, listen to chest and expansion, percussion trachea position. C - HR, BP, 2 wide-bore cannulas, CRT, FBC, group and save, coag, CRP, LFTs, U+Es D - GCS, BM E - temp and examine rectum and PR to assess if blood, palpated abdomen for pan
94
You are bleeped to see an 82 year old lady with significant PR bleeding in A&E. She usually lives in a nursing home and presents with a three day history of bloody diarrhoea. Her only past medical history is atrial fibrillation. Her observations are: Respiratory rate 25 breaths per minute Oxygen saturations 93% on air Blood pressure 110/65 Heart Rate 100 bpm Temperature 37.5 degrees Celsius What are the likely causes of her symptoms?
Could be - Ischaemic bowel Haemorrhoids with poor bleeding control due to DOAC for AF Diverticular disease Gastroenteritis Anal fissure/fistula UC/Crohns Angiodysplasia
95
You are bleeped to see an 82 year old lady with significant PR bleeding in A&E. She usually lives in a nursing home and presents with a three day history of bloody diarrhoea. Her only past medical history is atrial fibrillation. Her observations are: Respiratory rate 25 breaths per minute Oxygen saturations 93% on air Blood pressure 110/65 Heart Rate 100 bpm Temperature 37.5 degrees Celsius What else would you like to know from the history?
Duration and onset of symptoms Description of pain if any Any nausea or vomiting Amount of blood Anyone unwell in nursing home PMH of bowel issues Meds that may exacerbate bleeding Social hx to get baseline and gauge escalation plan
96
You are bleeped to see an 82 year old lady with significant PR bleeding in A&E. She usually lives in a nursing home and presents with a three day history of bloody diarrhoea. Her only past medical history is atrial fibrillation. Her observations are: Respiratory rate 25 breaths per minute Oxygen saturations 93% on air Blood pressure 110/65 Heart Rate 100 bpm Temperature 37.5 degrees Celsius What further investigations would you like?
I would request a full blood count (to assess for anaemia and signs of infection), CRP (to assess for inflammation), clotting profile (to assess for coagulopathy) as well as urea and electrolytes and liver function tests. Given the bleeding it may also be prudent to obtain a group and save sample (with multiple samples at different times if indicated by the trust policy), should blood products be required. I would also obtain an arterial blood gas. ECG PR exam Stool culture May need CT angiogram for ischaemic bowel however I would discuss this with seniors.
97
You are bleeped to see an 82 year old lady with significant PR bleeding in A&E. She usually lives in a nursing home and presents with a three day history of bloody diarrhoea. Her only past medical history is atrial fibrillation. Her observations are: Respiratory rate 25 breaths per minute Oxygen saturations 93% on air Blood pressure 110/65 Heart Rate 100 bpm Temperature 37.5 degrees Celsius Her arterial blood gas results are now available: pH 7.28 PaO2 10.5kPa PaC02 4kPa Bicarbonate 15 Base excess -4 Lactate 4.5. How would you manage her?
Metabolic acidosis with raised lactate. Concern regarding perfusion injury. Would give IVT and abx and expedite CT angiogram and discuss with general surgery. I would also stop metformin if prescribed and prescribe IVT
98
You are bleeped to see an 82 year old lady with significant PR bleeding in A&E. She usually lives in a nursing home and presents with a three day history of bloody diarrhoea. Her only past medical history is atrial fibrillation. Her observations are: Respiratory rate 25 breaths per minute Oxygen saturations 93% on air Blood pressure 110/65 Heart Rate 100 bpm Temperature 37.5 degrees Celsius Her arterial blood gas results are now available: pH 7.28 PaO2 10.5kPa PaC02 4kPa Bicarbonate 15 Base excess -4 Lactate 4.5. Who else would you involve?
Med reg General surgery team as likely ischaemic bowel Discuss with ED consultant for place in resus as critically unwell Discuss with patient the severity of illness and discuss TEP with her and family. Possible crit care discussion if escalation is appropriate.
99
You are bleeped to see an 82 year old lady with significant PR bleeding in A&E. She usually lives in a nursing home and presents with a three day history of bloody diarrhoea. Her only past medical history is atrial fibrillation. Her observations are: Respiratory rate 25 breaths per minute Oxygen saturations 93% on air Blood pressure 110/65 Heart Rate 100 bpm Temperature 37.5 degrees Celsius Her arterial blood gas results are now available: pH 7.28 PaO2 10.5kPa PaC02 4kPa Bicarbonate 15 Base excess -4 Lactate 4.5. You now have one minute to handover the patient in this scenario to your registrar/consultant as if you were at the Acute Medical Handover.
Situation: I am concerned about this 82 year old lady with a raised lactate and metabolic acidosis secondary to hypoperfusion as a result of a significant PR-bleed. Background: She has a history of persistent pr-bleeding and is also known to have AF. Her drug history is not known. Assessment: Fluid resuscitation has been started and a group and save ad crossmatch has been urgently sent to the lab. She is tachycardiac and hypotensive. Recommendations: I am worried that this patient could quickly deteriorate. I have recommended that she is moved to the Resus department in A+E where she can be closely monitored. She needs an urgent senior review. I have asked the team to urgently request her drug history from the nursing home. If she is on anticoagulation for her AF she may need a reversal agent or discussion with haematology. In the event of a further deterioration she should be given O-negative blood from the transfusion fridge pending her formal cross-matched sample. Given the presentation and her continued deterioration, she may benefit from being reviewed by the general surgical team.
100
A 30-year-old woman presents with a three-day history of shortness of breath, cough, pleuritic chest pain and fever. She has had several episodes of haemoptysis, which she describes as blood-stained sputum. She has a background of a previous deep vein thrombosis two years ago whilst on the combined oral contraceptive pill. She is a non-smoker. How would you manage this patient?
How would you manage this patient? I would attend to this patient in an A to E approach A – I would ensure her airway is patent and she is speaking. If there was an issue here I would manage this with airway adjuncts and anaesthetic input before moving on. B - RR, sats, chest expansion, tracheal position, percussion, CXR, ABG if desaturating C - CRT, HR, BP, Cannula - fbc, crp, lfts, U+Es, D-dimer, lactate, ECG, listen to heart sounds D - GCS and BMs E - temp and expose for rashes, DVT/Calf swellings Once complete and stable, would take hx from patient
101
A 30-year-old woman presents with a three-day history of shortness of breath, cough, pleuritic chest pain and fever. She has had several episodes of haemoptysis, which she describes as blood-stained sputum. She has a background of a previous deep vein thrombosis two years ago whilst on the combined oral contraceptive pill. She is a non-smoker. Her observations are: oxygen saturations 94% on room airA RR 32, HR 110, BP 90/60, T 38.0 What investigations would you do?
Significant concern here regarding chest sepsis. Would complete sepsis 6. Would consider fluid bolus and ECG as tachycardic and lw BP. CXR for consolidation and do wells score for PE. Sputum culture, COVID swab
102
A 30-year-old woman presents with a three-day history of shortness of breath, cough, pleuritic chest pain and fever. She has had several episodes of haemoptysis, which she describes as blood-stained sputum. She has a background of a previous deep vein thrombosis two years ago whilst on the combined oral contraceptive pill. She is a non-smoker. Her observations are: oxygen saturations 94% on room airA RR 32, HR 110, BP 90/60, T 38.0 . Here are the results of some investigations. Bloods Hb 130 g/dL, WBC 16 x 10^9/L, Neut 13 x 10^9/L, Plt 500 x 10^9/L, CRP 75, Urea 17 mmol/L, D-dimer 1.76 (0.1-0.45mg/L), CXR: Hazy opacification in the left lower zone, ECG Sinus tachycardia. Please interpret them.
Stable Hb Raised WCC and neutrophils suggestive of bacterial infection. Platelets mildly raised suggestive of acute phase reactant CRP raised suggestive of bacterial infection Raised urea - may be clinically dry D-dimer is raised - could suggest PE or raised due to infeciton as NOT specific to PE. CXR - CAP Sinus tachycardia as septic Would discuss elevated D-dimer with med reg to determine if need to investigate for PE
103
What is your approach to using and interpreting D-dimers?
A D-dimer may not initially be helpful as it has a low specificity of 45% BUT when used with modified wells score - higher degree of specificity and thus if raised wells score - consider CTPA
104
A 30-year-old woman presents with a three-day history of shortness of breath, cough, pleuritic chest pain and fever. She has had several episodes of haemoptysis, which she describes as blood-stained sputum. She has a background of a previous deep vein thrombosis two years ago whilst on the combined oral contraceptive pill. She is a non-smoker. Her observations are: oxygen saturations 94% on room airA RR 32, HR 110, BP 90/60, T 38.0 . Here are the results of some investigations. Bloods Hb 130 g/dL, WBC 16 x 10^9/L, Neut 13 x 10^9/L, Plt 500 x 10^9/L, CRP 75, Urea 17 mmol/L, D-dimer 1.76 (0.1-0.45mg/L), CXR: Hazy opacification in the left lower zone, ECG Sinus tachycardia. The patient does not respond to initial treatment, what will you do?
I would inform my registrar and ask for their urgent input, whilst also contacting ITU to ask for their review and input. It may be that the patient would require admission to ITU. They would provide closer monitoring, vasopressor support and NIV if required.
105
A 30-year-old woman presents with a three-day history of shortness of breath, cough, pleuritic chest pain and fever. She has had several episodes of haemoptysis, which she describes as blood-stained sputum. She has a background of a previous deep vein thrombosis two years ago whilst on the combined oral contraceptive pill. She is a non-smoker. Her observations are: oxygen saturations 94% on room airA RR 32, HR 110, BP 90/60, T 38.0 . Here are the results of some investigations. Bloods Hb 130 g/dL, WBC 16 x 10^9/L, Neut 13 x 10^9/L, Plt 500 x 10^9/L, CRP 75, Urea 17 mmol/L, D-dimer 1.76 (0.1-0.45mg/L), CXR: Hazy opacification in the left lower zone, ECG Sinus tachycardia. How would you demonstrate to your senior colleagues the severity of this patient's presentation?
CURB 65 score - scores 3
106
A 30-year-old woman presents with a three-day history of shortness of breath, cough, pleuritic chest pain and fever. She has had several episodes of haemoptysis, which she describes as blood-stained sputum. She has a background of a previous deep vein thrombosis two years ago whilst on the combined oral contraceptive pill. She is a non-smoker. Her observations are: oxygen saturations 94% on room airA RR 32, HR 110, BP 90/60, T 38.0 . Here are the results of some investigations. Bloods Hb 130 g/dL, WBC 16 x 10^9/L, Neut 13 x 10^9/L, Plt 500 x 10^9/L, CRP 75, Urea 17 mmol/L, D-dimer 1.76 (0.1-0.45mg/L), CXR: Hazy opacification in the left lower zone, ECG Sinus tachycardia. What would you say to the patient if they were in front of you now?
I would first ensure privacy if I could by drawing curtains. I would introduce myself and my role. I would ask the patient if there was anyone else she wanted to be there such as family over friends. I would ask her what she understood so far of what had been happening in hospital and what we were treating her for. I would then explain in simple language that we were treating her for a bad chest infection and that she needs to stay in hospital, and possibly go to ITU for more intensive care. I would explain that she should start to feel better in the coming days. We would be treating her with antibiotics and fluids through the vein to help treat the infection. I would ask her to explain this back to me and ask if she had any questions. I would ensure she felt comfortable to ask questions of staff at any point during the admission. I would ask if she wanted me to call anyone to inform them.
107
A 30-year-old woman presents with a three-day history of shortness of breath, cough, pleuritic chest pain and fever. She has had several episodes of haemoptysis, which she describes as blood-stained sputum. She has a background of a previous deep vein thrombosis two years ago whilst on the combined oral contraceptive pill. She is a non-smoker. Her observations are: oxygen saturations 94% on room airA RR 32, HR 110, BP 90/60, T 38.0 . Here are the results of some investigations. Bloods Hb 130 g/dL, WBC 16 x 10^9/L, Neut 13 x 10^9/L, Plt 500 x 10^9/L, CRP 75, Urea 17 mmol/L, D-dimer 1.76 (0.1-0.45mg/L), CXR: Hazy opacification in the left lower zone, ECG Sinus tachycardia. Not improving, requiring ITU input Give an SBAR
Situation: This is a 30 year old woman being treated for sepsis, secondary to a community acquired pneumonia. Background: She presented with a three day history of breathlessness, cough, pleuritic chest pain and fever. She has a past medical history of DVT, whilst taking the oral contraceptive pill, two years ago, but is otherwise fit and well.The symptoms have been present for the last three days and she has had several episodes of haemoptysis. Assessment: She was hypoxic, hypotensive, febrile and tachycardic on assessment, so the sepsis six has been initiated as per local guidelines as per local guidelines. Her chest x-ray shows evidence of left lower zone consolidation, supportive of pneumonia.pneumonia. Her D-dimer is noted to be elevated but this is likely in the context of her infection, rather than a PE. She has a CURB-65 score of 3. Recommendations: This lady needs to be admitted to the acute medical ward for ongoing IV antibiotics. She needs a senior review to determine further investigations including the need for a CTPA at this point, given her history of previous DVT and elevated D-dimer. Given her CURB-65 score I would recommend that this lady is reviewed early by the ITU team given her increased mortality.
108
How would you define sepsis and septic shock?
life-threatening organ dysfunction due to a dysregulated host response to infection. Septic shock and sepsis (despite adequate volume resuscitation) both consist of: Persistent hypotension requiring vasopressors to maintain MAP greater than or equal to 65 mm Hg, and Lactate greater than or equal to 2 mmol/L
109
Are there any risk factors you can think of that would make a patient more likely to develop sepsis?
Key risk factors that would make you more susceptible to sepsis include: underlying malignancy, age older than 65 years, immunocompromise, haemodialysis, alcoholism, and diabetes mellitus.
110
Do you know what the prognosis for someone with sepsis is?
The prognosis in patients with sepsis and septic shock is guarded at best. The mortality rate from sepsis has been estimated in a number of studies to be between 28% and 50%.
111
You are the medical SHO oncall overnight. The medical team have been fastbleeped to A&E resus by the nurse in charge to see a 58 year old man, who self discharged this morning. He has been brought back in by an ambulance crew who found him drowsy and confused, with reduced responsiveness at a bus stop. The medical registrar is busy with an arrest call and has asked you to review the patient and call him if there are any problems. His GCS is 9/15 (E2, V3, M4). The man has no fixed abode and was admitted 3 days ago with a similar presentation. He has no known medical history and has no known next of kin. He had remained in hospital due to social reasons before his self discharge. He smells strongly of alcohol and there is evidence that he may have vomited. On your way down to review the patient, what differential diagnoses would you consider?
Opioids, alcohol Head trauma Low BMs Stroke Seizure Aspiration
112
You are the medical SHO oncall overnight. The medical team have been fastbleeped to A&E resus by the nurse in charge to see a 58 year old man, who self discharged this morning. He has been brought back in by an ambulance crew who found him drowsy and confused, with reduced responsiveness at a bus stop. The medical registrar is busy with an arrest call and has asked you to review the patient and call him if there are any problems. His GCS is 9/15 (E2, V3, M4). The man has no fixed abode and was admitted 3 days ago with a similar presentation. He has no known medical history and has no known next of kin. He had remained in hospital due to social reasons before his self discharge. He smells strongly of alcohol and there is evidence that he may have vomited. How would you initially assess and manage this patient?
ABCDE A - would have concerns here. May need triple airway manouvure and trial of oropharyngeal airway. If tolerating then sign of ITU input likely needed. B - RR, sats, chest expansion, trachea central, good air entry bilaterally, give o2 if required, CXR if ?aspiration C - CRT, HR, BP, Bloods - CRP, LFTs, U+Es, FBC, lactate VBG, ECG to assess QT etc D - GCS, BMs E - rashes, head injuries, bruises, bony injury, temp
113
What are the common causes of hypoglycaemia?
Too much insulin Sulfonlyureas Not enough food Alcohol Beta blockers can cause hypos Adrenal crisis
113
You are the medical SHO oncall overnight. The medical team have been fastbleeped to A&E resus by the nurse in charge to see a 58 year old man, who self discharged this morning. He has been brought back in by an ambulance crew who found him drowsy and confused, with reduced responsiveness at a bus stop. The medical registrar is busy with an arrest call and has asked you to review the patient and call him if there are any problems. His GCS is 9/15 (E2, V3, M4). The man has no fixed abode and was admitted 3 days ago with a similar presentation. He has no known medical history and has no known next of kin. He had remained in hospital due to social reasons before his self discharge. He smells strongly of alcohol and there is evidence that he may have vomited. You are handed an ABG on arrival to the resus department, the results of which are as follows:pH 7.36 pCO2 5.6 pO2 35 Na 130 K 5.7 Glu 1.2 HCO3- 25. What is the cause of the presentation and what would you do now?
The cause of the patient’s presentation is hypoglycaemia. I would make sure bloods have been taken and then start 100mls of 20% glucose intravenously. If IV access is impossible, I would give 1mg of glucagon IM. However, IM glucagon may not work if alcoholic If hypoglycaemia is the cause of the unconsciousness then the patient should start to show signs of improvement within 10 minutes. Further IV glucose should not be given before repeating the blood glucose. Once the patient is alert enough, I would give them a carbohydrate rich food.
114
You are the medical SHO oncall overnight. The medical team have been fastbleeped to A&E resus by the nurse in charge to see a 58 year old man, who self discharged this morning. He has been brought back in by an ambulance crew who found him drowsy and confused, with reduced responsiveness at a bus stop. The medical registrar is busy with an arrest call and has asked you to review the patient and call him if there are any problems. His GCS is 9/15 (E2, V3, M4). The man has no fixed abode and was admitted 3 days ago with a similar presentation. He has no known medical history and has no known next of kin. He had remained in hospital due to social reasons before his self discharge. He smells strongly of alcohol and there is evidence that he may have vomited. You are handed an ABG on arrival to the resus department, the results of which are as follows:pH 7.36 pCO2 5.6 pO2 35 Na 130 K 5.7 Glu 1.2 HCO3- 25. In the A&E department you are unable to give IV glucose as access is incredibly difficult. You ask the nurse to give IM glucagon however there is no response in blood glucose. What is the cause of this patient’s hypoglycaemia?
Shows that it is likely low BMs due to alcohol thus wont work as no hepatic stores of glucose
115
What information should be given to patients with diabetes who drink alcohol?
Diabetic patients with episodes of hypoglycaemia need education in nutrition, checking glucose levels at home, and early signs and symptoms of hypoglycaemia. Recognition of early symptoms is paramount for self-treatment. DVLA - must inform them. Shuld not drive if BM less than 5
116
You are the medical SHO oncall overnight. The medical team have been fastbleeped to A&E resus by the nurse in charge to see a 58 year old man, who self discharged this morning. He has been brought back in by an ambulance crew who found him drowsy and confused, with reduced responsiveness at a bus stop. The medical registrar is busy with an arrest call and has asked you to review the patient and call him if there are any problems. His GCS is 9/15 (E2, V3, M4). The man has no fixed abode and was admitted 3 days ago with a similar presentation. He has no known medical history and has no known next of kin. He had remained in hospital due to social reasons before his self discharge. He smells strongly of alcohol and there is evidence that he may have vomited. You are handed an ABG on arrival to the resus department, the results of which are as follows:pH 7.36 pCO2 5.6 pO2 35 Na 130 K 5.7 Glu 1.2 HCO3- 25. In the A&E department you are unable to give IV glucose as access is incredibly difficult. You ask the nurse to give IM glucagon however there is no response in blood glucose. You now have one minute to handover the patient in this scenario to your registrar/consultant as if you were at the acute medicine handover.
Situation: I have a 58 year old man who has presented with a low GCS (9/15) secondary to hypoglycaemia. The hypoglycaemia has been caused by acute excessive alcohol consumption. Background: This patient self-discharged from hospital this morning having presented similarly three days ago. Assessment: There has been no response to IM glucagon and IV access has not been possible. He remains drowsy with a low GCS and I am concerned about his airway given his level of arousal. Recommendations: He needs urgent IV access. I will inform the ITU SPR about this man as they may be able to help with securing IV access and they will need to assess him from an airway point of view. He may need to be managed initially in a high dependency unit. He will need input in the long term to review his alcohol consumption and ways to address this.
117
A 19 year old woman attends A&E after being referred by her GP. She has been complaining of a headache for the last two days. She has been feeling increasingly unwell with it and is now vomiting. She has recently started at university and is normally fit and well. Her observations are as follows: Temperature: 38.6 degrees Heart Rate: 115 Respiratory Rate: 26 Saturations: 98% on room air Blood pressure: 110/63 You have been asked by the medical registrar to go and review the patient. What is your initial differential diagnosis based on the above presentation?
From the offset, my worry would be meningitis. Especially since she is young, septic, headaches and just started university. I would be most worried about meningococcal meningitis. Other things would be a SAH, cavernous venous sinus thrombosis, encephalitis, flu/covid, CAP - legionella and mycoplasma can cause headache.
118
A 19 year old woman attends A&E after being referred by her GP. She has been complaining of a headache for the last two days. She has been feeling increasingly unwell with it and is now vomiting. She has recently started at university and is normally fit and well. Her observations are as follows: Temperature: 38.6 degrees Heart Rate: 115 Respiratory Rate: 26 Saturations: 98% on room air Blood pressure: 110/63 You have been asked by the medical registrar to go and review the patient. How would you assess and manage this patient?
First I would start with an A-E Assessment. A - Is the patient talking? Are they able to maintain their own airway? B - RR, Sats, chest expansion, work of breathing, auscultation and trachea positioning, would not give supplemental oxygen at this point C - BP, HR, CRT, Heart sounds, ECG as tachycardic, cannula - CRP, LFTs, U+Es, FBC, BC, Lactate, IV ceftraxone if no allergies, IVT D - BM and GCS and assess pupils and for neck stiffness E - Temp. Checking the abdomen and limbs for signs of infection or cause of the presenting complaint. I would look for any evidence of a petechial rash.
119
A 19 year old woman attends A&E after being referred by her GP. She has been complaining of a headache for the last two days. She has been feeling increasingly unwell with it and is now vomiting. She has recently started at university and is normally fit and well. Her observations are as follows: Temperature: 38.6 degrees Heart Rate: 115 Respiratory Rate: 26 Saturations: 98% on room air Blood pressure: 110/63 You have been asked by the medical registrar to go and review the patient. What investigations would you like to do?
Bloods - CRP, LFTs, U+Es, FBC, BC, Lactate, Clotting for LP, glucose, Mg, calcium and phosphate Organise an LP. May be able to do this if competent as IMT or under direct guidance from senior. Consider CT head to R/O bleed. Would discuss with senior but important given patients presentation. I would send the sample for MC+S, cell count and gram stain, in addition to biochemistry (glucose and protein) and viral PCR. Urine dip, ECG and CXR for completeness
120
A 19 year old woman attends A&E after being referred by her GP. She has been complaining of a headache for the last two days. She has been feeling increasingly unwell with it and is now vomiting. She has recently started at university and is normally fit and well. Her observations are as follows: Temperature: 38.6 degrees Heart Rate: 115 Respiratory Rate: 26 Saturations: 98% on room air Blood pressure: 110/63 You have been asked by the medical registrar to go and review the patient. What other signs and symptoms would make you think meningitis?
Symptoms - headaches, neck stiffness, fevers, photophobia, confusion/seizures if associated encephalitis, petechial rash if meningococcal, recent infection in sinuses/chest/ear as source Signs - foreign travel, starting university, unvaccinated
121
A 19 year old woman attends A&E after being referred by her GP. She has been complaining of a headache for the last two days. She has been feeling increasingly unwell with it and is now vomiting. She has recently started at university and is normally fit and well. Her observations are as follows: Temperature: 38.6 degrees Heart Rate: 115 Respiratory Rate: 26 Saturations: 98% on room air Blood pressure: 110/63 You have been asked by the medical registrar to go and review the patient. Do you know any bacteria that are commonly associated with bacterial meningitis?
Strep pneumonia (commonest in middle age), Listeria (common in old and young), coxsackie virus, group B strep in young, E-coli, Niscerria menigntitis, TB
122
A 19 year old woman attends A&E after being referred by her GP. She has been complaining of a headache for the last two days. She has been feeling increasingly unwell with it and is now vomiting. She has recently started at university and is normally fit and well. Her observations are as follows: Temperature: 38.6 degrees Heart Rate: 115 Respiratory Rate: 26 Saturations: 98% on room air Blood pressure: 110/63 You have been asked by the medical registrar to go and review the patient. What antibiotics would you use?
If rash - benzylpenicillin If no rash - ceftriaxone If penicillin allergy then cant use cephlasporins so would give chloromphenicol or vancomycin
123
A 19 year old woman attends A&E after being referred by her GP. She has been complaining of a headache for the last two days. She has been feeling increasingly unwell with it and is now vomiting. She has recently started at university and is normally fit and well. Her observations are as follows: Temperature: 38.6 degrees Heart Rate: 115 Respiratory Rate: 26 Saturations: 98% on room air Blood pressure: 110/63 You have been asked by the medical registrar to go and review the patient. Is a CT head needed before LP?
LP can be performed in patients with suspected meningitis without a CT head. However, this is only advised in cases of simple meningitis – i.e. not obviously septicaemic, no focal neurological signs and NO decrease in consciousness.
124
A 19 year old woman attends A&E after being referred by her GP. She has been complaining of a headache for the last two days. She has been feeling increasingly unwell with it and is now vomiting. She has recently started at university and is normally fit and well. Her observations are as follows: Temperature: 38.6 degrees Heart Rate: 115 Respiratory Rate: 26 Saturations: 98% on room air Blood pressure: 110/63 You have been asked by the medical registrar to go and review the patient. What would you expect to see in the CSF to suggest bacterial meningitis? And how would someone with VIRAL meningitis differ?
Bacterial - Turbid, low glucose, elevated protein, mainly neutrophils Viral - clear, normal glucose, high protein, mostly lymphocytes
125
Who should you notify about cases of bacterial meningitis?
Close contacts need contact if confirmed meningococcal. Such as friends and housemates - need ciprofloxacin or rifampicin. Would also have to inform public health and the university.
126
What would be in your differential list for a patient with a purpuric rash and a fever?
Top of the list MUST be menincococcal sepsis Others - Thrombotic Thrombocytopenic Purpura, DIC, Vasculitis such as henoch sherloins, Viral haemorrhagic fevers
127
A 19 year old woman attends A&E after being referred by her GP. She has been complaining of a headache for the last two days. She has been feeling increasingly unwell with it and is now vomiting. She has recently started at university and is normally fit and well. Her observations are as follows: Temperature: 38.6 degrees Heart Rate: 115 Respiratory Rate: 26 Saturations: 98% on room air Blood pressure: 110/63 You have been asked by the medical registrar to go and review the patient. Give an SBAR
Situation: This is a 19 year old female university student who presented with symptoms and signs consistent with bacterial meningitis. She is pyrexial and tachycardic. Background: She is normally fit and well and I am not aware of any predisposing factors. Assessment: I have assessed the patient and full investigations as part of the sepsis work up have been sent, including blood cultures. A lumbar puncture will be required to help with the diagnosis. Recommendations: This lady will require urgent treatment with broad spectrum antibiotics to cover CNS infections as per local guidelines. Ideally these will be started pending the LP, however, if this is delayed then antibiotics should be given. She requires a review with one of the senior clinical team as she could deteriorate quickly. If bacterial meningitis is confirmed then local public health services will need to be informed.
128